Bold Face - City Transportation

This topic has expert replies
Legendary Member
Posts: 1799
Joined: Wed Dec 24, 2008 3:03 am
Thanked: 36 times
Followed by:2 members

Bold Face - City Transportation

by goelmohit2002 » Mon Aug 10, 2009 4:31 am
Hi All,

Can someone please tell what is the conclusion of the following argument:

1. The city government should invest surplus funds in improving the city's transportation network.......or
2. If the city does not make changes soon to the network, it will see many of its prized industries relocate to more convenient cities and, as a result, the city's financial health will be jeopardized.


===============================================


The city government should invest surplus funds in improving the city's transportation network. Most of the network was put in place at a time when the city was much smaller in both area and population. The subway system is outdated and understaffed. The buses rarely run on schedule and their routes are inconvenient. If the city does not make changes soon to the network, it will see many of its prized industries relocate to more convenient cities and, as a result, the city's financial health will be jeopardized.

In the argument above, the two portions in boldface play which of the following roles?

A. The first is an explanation of a current state of affairs; the second is a prediction based on that state of affairs.

B. The first is a statement of fact in opposition to the author's conclusion; the second is that conclusion.

C. The first emphasizes an existing problem; the second offers a proposal to solve that problem.

D. The first is information the author suggests has been overlooked in the situation at hand; the second describes that situation.

E. The first is a justification of an impending problem; the second describes the consequences of that problem.

OA = A

Senior | Next Rank: 100 Posts
Posts: 37
Joined: Fri Jul 31, 2009 6:41 am
Thanked: 1 times

by acenikk » Mon Aug 10, 2009 4:50 am
It is A. Although it is not a very good answer, but it is the best among all the choices. It is not a very good answer as it suggests that the prediction in the second bold statement is based on the current state of affairs, which according to statement 1 is that the network was put in place at a very older time. I guess the prediction is based on rather that the subway system is outdated and the buses does not run on schedule.

The conclusion is that the city government should invest surplus funds in improving the city's transportation network.
Last edited by acenikk on Mon Aug 10, 2009 4:55 am, edited 2 times in total.

Legendary Member
Posts: 1799
Joined: Wed Dec 24, 2008 3:03 am
Thanked: 36 times
Followed by:2 members

by goelmohit2002 » Mon Aug 10, 2009 4:53 am
acenikk wrote:The conclusion is that the city government should invest surplus funds in improving the city's transportation network.
Thanks. I am also confused with this.

The OE says otherwise.....it says that...the conclusion is #2...i.e.

"If the city does not make changes soon to the network, it will see many of its prized industries relocate to more convenient cities and, as a result, the city's financial health will be jeopardized. "


Can someone please tell why it is not #1 but #2......

Legendary Member
Posts: 876
Joined: Thu Apr 10, 2008 8:14 am
Thanked: 13 times

by ketkoag » Mon Aug 10, 2009 8:35 am
goelmohit2002 wrote:
acenikk wrote:The conclusion is that the city government should invest surplus funds in improving the city's transportation network.
Thanks. I am also confused with this.

The OE says otherwise.....it says that...the conclusion is #2...i.e.

"If the city does not make changes soon to the network, it will see many of its prized industries relocate to more convenient cities and, as a result, the city's financial health will be jeopardized. "
please check the boldfaced above..
"As a result" indicates the definite conclusion....
first statement is not the main conclusion of the stimuli....

Legendary Member
Posts: 708
Joined: Sun Jun 01, 2008 4:59 am
Location: USA
Thanked: 13 times
Followed by:1 members

by niraj_a » Mon Aug 10, 2009 9:43 am
why not E guys? I didn't think that the bold statement 1 was current state of affairs since it talks about what was done in the past.

i'm mistaken somehow.

Legendary Member
Posts: 1799
Joined: Wed Dec 24, 2008 3:03 am
Thanked: 36 times
Followed by:2 members

by goelmohit2002 » Mon Aug 10, 2009 10:04 am
ketkoag wrote: please check the boldfaced above..
"As a result" indicates the definite conclusion....
first statement is not the main conclusion of the stimuli....
But IMO isn't "should" in is equally emphasing the point that author make.....shouldn't this be the "what" author want to communicate...

"The city government should invest surplus funds in improving the city's transportation network. ".....

What = City should invest more....
Why = Because network is outdated and if not updated, then companies will move.

Please tell what is the flaw in the above ?

Legendary Member
Posts: 1799
Joined: Wed Dec 24, 2008 3:03 am
Thanked: 36 times
Followed by:2 members

by goelmohit2002 » Mon Aug 10, 2009 10:08 am
niraj_a wrote:why not E guys? I didn't think that the bold statement 1 was current state of affairs since it talks about what was done in the past.

i'm mistaken somehow.
I was also confused with the same....but after reading the OE I found that impending = likely to happen in future....but this is not a future problem but the current problem....so wrong....

OE also said that(which I am not 100% convinced).....that in E.....it is possible consequences....not consequences.... so E is wrong....not sure do we have to see so much minutely :-)

Another reason that OE gave for kicking out E = FBF = justification is wrong.....I am not able to understand how using justification is wrong.....can someone please explain?

Legendary Member
Posts: 708
Joined: Sun Jun 01, 2008 4:59 am
Location: USA
Thanked: 13 times
Followed by:1 members

by niraj_a » Mon Aug 10, 2009 10:42 am
goelmohit2002 wrote:
niraj_a wrote:why not E guys? I didn't think that the bold statement 1 was current state of affairs since it talks about what was done in the past.

i'm mistaken somehow.
I was also confused with the same....but after reading the OE I found that impending = likely to happen in future....but this is not a future problem but the current problem....so wrong....

OE also said that(which I am not 100% convinced).....that in E.....it is possible consequences....not consequences.... so E is wrong....not sure do we have to see so much minutely :-)

Another reason that OE gave for kicking out E = FBF = justification is wrong.....I am not able to understand how using justification is wrong.....can someone please explain?
ah ok, that makes sense now. subtle yeah. is this LSAT?

Legendary Member
Posts: 1799
Joined: Wed Dec 24, 2008 3:03 am
Thanked: 36 times
Followed by:2 members

by goelmohit2002 » Mon Aug 10, 2009 1:09 pm
niraj_a wrote:
goelmohit2002 wrote:
niraj_a wrote:why not E guys? I didn't think that the bold statement 1 was current state of affairs since it talks about what was done in the past.

i'm mistaken somehow.
I was also confused with the same....but after reading the OE I found that impending = likely to happen in future....but this is not a future problem but the current problem....so wrong....

OE also said that(which I am not 100% convinced).....that in E.....it is possible consequences....not consequences.... so E is wrong....not sure do we have to see so much minutely :-)

Another reason that OE gave for kicking out E = FBF = justification is wrong.....I am not able to understand how using justification is wrong.....can someone please explain?
ah ok, that makes sense now. subtle yeah. is this LSAT?
This is from Manhattan....but can you please help me understand the other reasons that OE gave to kick out E...

Also please tell what do you think is the author's conclusion....since I am not 100% convinced by what OE say as conclusion.

Newbie | Next Rank: 10 Posts
Posts: 2
Joined: Fri Jul 24, 2009 9:54 pm

by xian1707 » Mon Aug 10, 2009 1:45 pm
goelmohit2002 wrote: Also please tell what do you think is the author's conclusion....since I am not 100% convinced by what OE say as conclusion.
I myself kick out choice E because I think E can not be consequense. It is a condition sentence in which the event has not happened yet.

GMAT/MBA Expert

User avatar
GMAT Instructor
Posts: 2228
Joined: Wed Dec 27, 2006 3:28 pm
Location: Montreal, Canada
Thanked: 639 times
Followed by:694 members
GMAT Score:780

by Stacey Koprince » Fri Aug 14, 2009 9:57 am
Received a PM asking me to respond.

1) gov't should invest funds in transport network
2) network is really old, when city was smaller
3) subway outdated
4) buses bad
5) if no changes are made, then companies will move away, and the city will go downhill

I think you could argue that statements 1 and 5 are both conclusions: one's about what we "should" do and the other is about what will happen if we don't do what we "should" do. You'll notice that the answer doesn't change either way.

I agree with acenikk that the correct answer is not worded as well as it should be.

The problem with E is the first half of the answer: "justification of an impending problem." What is described in the first boldface is not an "impending" problem - the actions have already occurred. It is a problem now, because the city has grown, but the statement itself does not reflect an "impending (coming, future) problem."

Also "justification" means to try to prove or show that something is correct. The statement by itself is not trying to prove something correct. It's just stating a fact. (other parts of the text are trying to justify, yes, but this one is just a fact, and this part of the answer is supposed to refer ONLY to the first boldface.)

Put the two together: justification of an impending problem. Trying to show that an impending problem is correct. The whole argument is trying to justify a solution to an existing problem, yes. But the first boldface is not trying to justify a future problem.
Please note: I do not use the Private Messaging system! I will not see any PMs that you send to me!!

Stacey Koprince
GMAT Instructor
Director of Online Community
Manhattan GMAT

Contributor to Beat The GMAT!

Learn more about me

Master | Next Rank: 500 Posts
Posts: 303
Joined: Sat Aug 22, 2015 10:23 am

by joseph32 » Sun May 15, 2016 9:15 pm
Option E looks good than other answers.